Download as pdf or txt
Download as pdf or txt
You are on page 1of 18

1 Candidate Name

PDG

ANDERSON JUNIOR COLLEGE

2011 JC2 Preliminary Examination

PHYSICS
Higher 2 Paper 1 Multiple Choice

9646/01
Wednesday 21 September 2011 1 hour 15 minutes

Additional materials: Multiple Choice Answer Sheet

READ THESE INSTRUCTIONS FIRST Write in soft pencil. Write your name and PDG in the spaces at the top of this page. Write your name, PDG, NRIC and shade the 7 digits of your NRIC/FIN in soft pencil on the Multiple Choice Answer Sheet. There are forty questions on this paper. Answer all questions. For each question there are four possible answers A, B, C and D. Choose the one you consider correct and record your choice in soft pencil on the separate Multiple Choice Answer Sheet. Each correct answer will score one mark. A mark will not be deducted for a wrong answer. Any rough working should be done in this question paper.

This document consists of 18 printed pages. 9646/01/AJC2011Prelim [Turn Over

2 Data speed of light in free space, permeability of free space, permittivity of free space, c = 3.00 x 108 m s-1

0 = 4 x 10-7 H m-1 0 = 8.85 x 10-12 F m-1


(1/(36)) x 10-9 F m-1

elementary charge, the Planck constant, unified atomic mass constant, rest mass of electron, rest mass of proton, molar gas constant, the Avogadro constant, the Boltzmann constant, gravitational constant, acceleration of free fall,

e = 1.60 x 10-19 C h = 6.63 x 10-34 J s u = 1.66 x 10-27 kg me = 9.11 x 10-31 kg mp = 1.67 x 10-27 kg R = 8.31 J K-1 mol-1 NA = 6.02 x 1023 mol-1 k = 1.38 x 10-23 J K-1 G = 6.67 x 10-11 N m2 kg-2 g = 9.81 m s-2

9646/01/AJC2011Prelim

3 Formulae uniformly accelerated motion,

v2 = u2 + 2as work done on/by a gas, hydrostatic pressure, gravitational potential, displacement of particle in s.h.m., velocity of particle in s.h.m., W = pV p = gh

s = ut + 1 at 2

=-

Gm r

x = x0 sin t v = v0 cos t

v = x 0 x 2
mean kinetic energy of a molecule of an ideal gas, resistors in series, resistors in parallel, electric potential, alternating current/voltage, transmission coefficient,

E=

3 kT 2

R = R1 + R2 + 1/R = 1/R1 + 1/R2 +


V = Q 4 0 r

x = x0 sin t T exp(-2kd)
where k =
8 2 m(U E ) h2

radioactive decay, decay constant,

x = x0exp(- t)

0.693 t1
2

9646/01/AJC2011Prelim

[Turn Over

4 1 A quantity X is to be determined from the equation

X=PQ
given that

P = 1.27 0.02 m. Q = 0.83 0.01 m.


What is the percentage uncertainty in X? A 0.4 % B 2% C 3% D 7%

Errors in measurement may be either systematic or random. Which of the following involves random errors? A not allowing for zero error on a voltmeter B not subtracting background count rate when determining the count rate from a radioactive source C stopping a stopwatch at the end of a race D using the value of g as 10 N kg-1 when calculating weight from mass

In order that a train can stop safely, it passes a signal showing a yellow light before it reaches another signal showing a red light. Drivers apply the brake at the yellow light and this results in a uniform deceleration to stop exactly at the red light. The distance between the red and yellow lights is x. What must be the minimum distance between the lights if the train speed is increased by 20%, without changing the deceleration of the trains? A 1.20x B 1.25x C 1.44x D 1.56x

9646/01/AJC2011Prelim

5 4 A student throws a ball vertically upwards. Upward velocities are taken as positive. The ball makes an elastic collision with the ceiling, rebounds and accelerates back to the students hand in a time of 1.2 s. Which graph best represents the acceleration of the ball from the moment it leaves the hand to the instant just before it returns to the hand? A B

The diagram shows two spherical masses approaching each other head-on at an equal speed u. One has mass 2m and the other has mass m.

Which diagram represents the situation after an elastic collision? A B

9646/01/AJC2011Prelim

[Turn Over

6 6 A box of mass 8.0 kg rests on a horizontal, rough surface. A string attached to the box passes over a smooth pulley and supports a 2.0 kg mass at its other end.

When the box is released, a friction force of 6.0 N acts on it. What is the acceleration of the box? A 1.4 m s-2 B 1.7 m s-2 C 2.0 m s-2 D 2.5 m s-2

The diagram shows a metal cube suspended from a spring balance before and during immersion in water.

A reduction in the balance reading occurs as a consequence of the immersion. Which statement is correct? A The balance reading will be further reduced if the cube is lowered further into the water. B The balance reading during immersion corresponds to the upthrust of the water on the cube. C The forces acting on the vertical sides of the cube contribute to the change in the balance reading. D The gravitational pull on the cube is unchanged by the immersion. 9646/01/AJC2011Prelim

7 8 A rigid L-shaped lever arm is pivoted at point P.

Three forces act on the lever arm, as shown in the diagram. What is the magnitude of the resultant moment of these forces about point P? A 15 Nm B 20 Nm C 35 Nm D 75 Nm

The diagram shows two identical vessels X and Y connected by a short pipe with a tap.

Initially, X is filled with water of mass m to a depth h, and Y is empty. When the tap is opened, water flows from X to Y until the depths of water in both vessels are equal. How much potential energy is lost by the water during this process? (g = acceleration of free fall) A 0 B

mgh 4

mgh 2

D mgh

9646/01/AJC2011Prelim

[Turn Over

8 10 Two trolleys are placed together on a horizontal runway with a compressed spring between them.

When they are released, the 2 kg trolley moves to the left at 2 m s-1. How much energy was stored in the spring? A 4J B 6J C 8J D 12 J

11

A mass on the end of a string is set in motion so that it describes a circle in a horizontal plane at a constant speed. The angle made by the string to the vertical is 40. The only forces acting on the mass are the tension T and its weight W. What is the ratio of the resultant force to the weight of the mass?

40

W
A 0.012 12 B 0.643 C 0.766 D 0.839

What is the speed of a geostationary satellite orbiting Earth? (mass of the Earth = 6.0 1024 kg) A 3.08 103 m s1 B 4.71 104 m s1 C 1.80 105 m s1 D 4.23 107 m s1

13

What is meant by the weight of an object? A B C D the gravitational field acting on the object the gravitational force acting on the object the mass of the object multiplied by gravity the objects mass multiplied by its acceleration

9646/01/AJC2011Prelim

9 14 The diagram shows points W, X, Y and Z on a line joining the centre of the Earth to the centre of the Moon. W and Z are on the surfaces of the Earth and Moon respectively. X is the mid-point between the Earth and the Moon. Y is a point where an object will not experience any resultant force.

Earth X Y

Moon W Z

A spacecraft moves between the Earth and the Moon along this line. Which transition would give the greatest change in gravitational potential energy of the spacecraft? A W to X 15 B W to Y C W to Z D Y to Z

The diagram shows a velocity-time graph for a mass moving up and down on the end of a spring. Taking velocity upward to be positive, which point represents the velocity of the mass when at the lowest point of its motion?

9646/01/AJC2011Prelim

[Turn Over

10 16 The diagram below shows the variation with frequency f of the amplitude x0 of the forced oscillation of a machine. x0 / cm

16 14

12 10

8 6

4 2 0

0.5

1.0

1.5

2.0

2.5

3.0

f / Hz

At resonance, calculate the maximum linear velocity and state whether the machine experience any damping. velocity / m s-1 A B C D 17 1.55 1.55 14.6 14.6 damping No Yes No Yes

Which phenomenon is associated with transverse waves but not longitudinal waves? A rarefaction B interference C polarisation D reflection

9646/01/AJC2011Prelim

11 18 A health inspector is measuring the intensity of a sound. Near a loudspeaker his meter records an intensity I. This corresponds to an amplitude A of the sound wave. At another position the meter gives an intensity reading of 2I. What is the corresponding sound wave amplitude? A A C 2A B 2A 2 19

D 4A

A wire of length L fixed at both ends is placed vertically between two magnets of opposite poles. A stationary wave is set up in the wire as shown in the diagram when an alternating voltage of frequency 50 Hz is applied across the wire.

Which of the following is false? The wire will not vibrate if the voltage is a direct source. The greatest amplitude is obtained when the wavelength of the stationary wave set up in wire is 2L. The wire would vibrate at 50 Hz, regardless of the strength of the magnetic field applied by the magnet. The wire will set up a stationary wave at when the frequency of the alternating voltage is reduced to 25 Hz.

A B

20

A parallel beam of white light is incident normally on a diffraction grating. It is noted that the second-order and third-order spectra partially overlap. Which wavelength in the third-order spectrum appears at the same angle as the wavelength of 600 nm in the second-order spectrum?
A 300 nm B 400 nm C 600 nm D 900 nm

21

When one junction X of a thermocouple is placed in melting ice and the other junction Y in steam at 100 C, the e.m.f. is 8.0 mV. Junction X is removed from the melting ice and is placed in a liquid bath at a constant temperature and junction Y is removed from the steam and placed in water at a constant temperature of 40 C. The e.m.f. is now -1.6 mV. What is the temperature of the bath on the centigrade scale of this thermocouple?
A 20 C B 60 C C 80 C 9646/01/AJC2011Prelim D 120 C [Turn Over

12 22

A sample of an ideal gas may (i) compress at constant temperature, or (ii) compress at constant pressure. The net flow of heat into the gas from the exterior is negative in each case positive in each case negative for (i) and positive for (ii) positive for (i) and negative for (ii)

A B C D 23

Two identical vessels contain two different gases X and Y with molecular mass mx and my respectively. The absolute temperature of gas X is twice that of gas Y and the root-meansquare speed of the molecules in X is three times that of the molecules in Y. What is the
m m

ratio of

.
B 2:9 C 9:2 D 3:2

X Y

A 2:3 24

The diagram shows a simple potentiometer circuit for measuring the e.m.f. of cell Q using a driver cell P and wire XY. However, no balance point could be found in this set-up. A balance point might be obtained by P

X galvanometer

Q
A B C D

replacing cell P by a cell of greater e.m.f. replacing wire XY by a wire with a greater cross-sectional area. adding a resistor in series with cell P. adding a resistor in series with cell Q.

9646/01/AJC2011Prelim

13 25

Two cells of e.m.f E1 and E2 with negligible internal resistance are connected with two variable resistors as shown in the diagram.

When the galvanometer deflection is zero, the resistances of the variable resistors are P

EE

1 2

and Q. What is the value of the ratio


A 26

?
C

P Q

P P +Q

Q P +Q

P +Q P

The circuit shown in the diagram below is connected to a power supply and the ammeter A1 reads 6.0 A.
2R R

4R

A1

3R

A2

Assuming that both ammeters have negligible resistance, what is the reading on ammeter A2?
A 27

2.3 A

4.5 A

6.0 A

13.5 A

Two parallel conducting plates are connected to a battery, one plate to the positive terminal and the other plate to the negative. The plate separation is gradually increased, the plates remaining connected to the battery. Which graph shows how the electric field E between the plates depends on the plate separation x?
A B C D

9646/01/AJC2011Prelim

[Turn Over

14 28

The diagram shows equipotential lines spaced 2.0 cm apart.

80 V
X

20 V

-40 V -100V

equipotential line 2.0 cm 2.0 cm 2.0 cm

The electric force exerted on a charge of 5.0 C when placed at X is 3.0 x 10-2 N to the right 3.0 x 10-2 N to the left 1.5 x 10-2 N to the right 1.5 x 10-2 N to the left

A B C D 29

A bent wire PQRST carrying a current I is placed in a magnetic field of flux density B as shown. QR is the shortest and ST is the longest. PQ and RS are equal in length and both are longer than QR.

R
B
I

The forces acting on PQ, QR, RS and ST are given by FPQ, FQR, FRS and FST respectively. Which of the following is true?
A B C D

FQR FRS and FPQ < FST FQR = FRS and FPQ = FST FQR > FRS and FPQ < FST FQR < FRS and FPQ = FST

9646/01/AJC2011Prelim

15 30

A particle of mass m and charge +q enters with speed v at right angles to a magnetic field of flux density B. The particle arrives at point P, a distance d from the slit as shown in the diagram below. Which of the following is a correct expression for d?

d
P

B v

4 2 m Bq

2mv Bq

2mv Bq

8mv Bq

31

A rectangular coil is rotating with constant angular velocity in a magnetic field. axis of rotation

Which one of the following statements is false?


A The maximum induced e.m.f. will increase if the frequency of rotation is increased. B The maximum induced e.m.f. will increase if the cross sectional area for the coil is increased. C The maximum induced e.m.f. will increase if the Earths magnetic field is in the same direction as the applied magnetic field. D The maximum induced e.m.f. will increase if the axis of rotation is parallel to the magnetic field.

9646/01/AJC2011Prelim

[Turn Over

16 32

A rectangular loop of wire enters a region of uniform magnetic field and leaves the field at a uniform speed v. The magnetic field is perpendicular to the plane of the loop of wire. Which graph best shows the variation with time t of the current I induced in the frame?

9646/01/AJC2011Prelim

17 33

The diagram shows the variation with time of a periodic current. The current of peak value I0 passes through a heater of resistance R. What is the mean power output of the heater?

Io I/A

t/ms

A 34

I 0 2R 4

I0 R 2

I0 R 2

I 0 2R

A metal surface in an evacuated tube is illuminated with monochromatic light causing the emission of photoelectrons which are collected at an adjacent electrode. If the experiment is to be repeated with light of half the intensity but the same wavelength, how will the photocurrent I and stopping potential V be affected?
A B C

I unchanged, V unchanged I unchanged, V halved I halved, V unchanged

D I halved, V halved

35

The following graph shows the spectrum of X-rays emitted from an X-ray tube. intensity

wavelength If the potential difference between the target and cathode is increased, which one of the following combinations represents a possible change in wavelength and intensity of the peaks? wavelength remain the same decrease remain the same decrease intensity increase remain the same remain the same increase

A B C D

9646/01/AJC2011Prelim

[Turn Over

18 36

A proton has a kinetic energy of 1.00 MeV. If its momentum is measured with an uncertainty of 1.00 %, what is the minimum uncertainty in its position?
A 1.95 1011 m B 4.57 1013 m C 9.08 1014 m D 6.59 1020 m

37

The resistance of a piece of pure silicon falls as the temperature rises. Which statement is true?
A The ratio of the positive to negative charge carriers increases. B The ratio of the positive to negative charge carriers decreases. C The charge carriers can move more easily at a higher temperature. D The total number of charge carriers increases with temperature.

38

The word LASER comes from the first letters of the words in the phrase Light Amplification by the Stimulated Emission of Radiation. What is meant by stimulated emission?
A B C

an electron from a higher level falling to a lower level a charged particle causing light to be emitted from an excited atom a charged particle being emitted from an atom as a result of a high energy photon hitting the atom. a photon causing another photon of the same frequency to be emitted from an excited atom

39

Radon

222 86

Rn decays by and emission to bismuth

214 83

Bi .

For the decay of each nucleus of radon, how many and particles are emitted?

particles
A B C D

particles
1 1 2 2

1 2 1 2

40

The activity from a radioactive source is found to fall by 0.875 of its initial activity in 210 s. What is the half-life of the source?
A 30 s B 70 s C 210 s D 1100 s

9646/01/AJC2011Prelim

You might also like